Question and Answers Forum

All Questions      Topic List

Algebra Questions

Previous in All Question      Next in All Question      

Previous in Algebra      Next in Algebra      

Question Number 180923 by Shrinava last updated on 19/Nov/22

Answered by aleks041103 last updated on 20/Nov/22

Forbenius: A,B,C∈M_n (F)  rk(abc)+rk(b)≥rk(ab)+rk(bc)  b=A  a=A^3   c=A  ⇒rk(AAA^3 )+rk(A)≥rk(A^3 A)+rk(AA)  ⇒rank(A^5 )+rank(A)≥rank(A^4 )+rank(A^2 )

$${Forbenius}:\:{A},{B},{C}\in{M}_{{n}} \left(\mathbb{F}\right) \\ $$$${rk}\left({abc}\right)+{rk}\left({b}\right)\geqslant{rk}\left({ab}\right)+{rk}\left({bc}\right) \\ $$$${b}={A} \\ $$$${a}={A}^{\mathrm{3}} \\ $$$${c}={A} \\ $$$$\Rightarrow{rk}\left({AAA}^{\mathrm{3}} \right)+{rk}\left({A}\right)\geqslant{rk}\left({A}^{\mathrm{3}} {A}\right)+{rk}\left({AA}\right) \\ $$$$\Rightarrow{rank}\left({A}^{\mathrm{5}} \right)+{rank}\left({A}\right)\geqslant{rank}\left({A}^{\mathrm{4}} \right)+{rank}\left({A}^{\mathrm{2}} \right) \\ $$

Commented by Shrinava last updated on 20/Nov/22

thank you dear professor cool

$$\mathrm{thank}\:\mathrm{you}\:\mathrm{dear}\:\mathrm{professor}\:\mathrm{cool} \\ $$

Terms of Service

Privacy Policy

Contact: info@tinkutara.com